Question on Linear Algebra (related to Affine Combination)












0












$begingroup$


The book I am using mentions this theorem:



$$Asubseteq Vtext{ is a flat} iff Atext{ is closed under affine combinations, that is}\ x_1,ldots,x_kin A, alpha_1,ldots,alpha_k,in F, alpha_1+cdots+ alpha_k=1Rightarrow alpha_1x_1+cdots+alpha_kx_kin A$$



The following question within the exercise then refers to this theorem (specifically the RHS of the above iff statement):




Let $ 1+1neq0$ in $F$.



Show that closure under affine combinations for $k=2$ implies closure under affine combinations for all $kgeq1$.






My approach so far has been to use Strong Induction.



Assuming the result holds for $kleq n$, we show it holds for $k=n+1$



consider $sum_{i=1}^{k+1}alpha_ix_i $



choose $lin{1,2,ldots, k+1}$ such that $sum_{i=1}^{k+1}alpha_ix_i=betasum_{i=1}^{l}frac{alpha_i}{beta}x_i+gammasum_{i={l+1}}^{k+1}frac{alpha_i}{gamma}x_i$



(Note: $beta = sum_{i=1}^{l}alpha_i neq0$ and $gamma = sum_{i=l+1}^{k+1}alpha_i neq0$)



Using induction hypothesis, $sum_{i=1}^{l}frac{alpha_i}{beta}x_iin A$ and $sum_{i={l+1}}^{k+1}frac{alpha_i}{gamma}x_iin A$. Call them $x$ and $y$ respectively.}



Now, $sum_{i=1}^{k+1}alpha_ix_i = beta x+ gamma y$



Since the result holds for $k=2$, $sum_{i=1}^{k+1}alpha_ix_iin A
$





My question is twofold. First, I never made use of the fact that $1+1neq0$ in F. What am I missing here? Second, how can I show that the condition in line 3 of my solution (when I introduce l and break the summation into two sums) can be assumed without loss of generality?



Any input would be appreciated.










share|cite|improve this question











$endgroup$

















    0












    $begingroup$


    The book I am using mentions this theorem:



    $$Asubseteq Vtext{ is a flat} iff Atext{ is closed under affine combinations, that is}\ x_1,ldots,x_kin A, alpha_1,ldots,alpha_k,in F, alpha_1+cdots+ alpha_k=1Rightarrow alpha_1x_1+cdots+alpha_kx_kin A$$



    The following question within the exercise then refers to this theorem (specifically the RHS of the above iff statement):




    Let $ 1+1neq0$ in $F$.



    Show that closure under affine combinations for $k=2$ implies closure under affine combinations for all $kgeq1$.






    My approach so far has been to use Strong Induction.



    Assuming the result holds for $kleq n$, we show it holds for $k=n+1$



    consider $sum_{i=1}^{k+1}alpha_ix_i $



    choose $lin{1,2,ldots, k+1}$ such that $sum_{i=1}^{k+1}alpha_ix_i=betasum_{i=1}^{l}frac{alpha_i}{beta}x_i+gammasum_{i={l+1}}^{k+1}frac{alpha_i}{gamma}x_i$



    (Note: $beta = sum_{i=1}^{l}alpha_i neq0$ and $gamma = sum_{i=l+1}^{k+1}alpha_i neq0$)



    Using induction hypothesis, $sum_{i=1}^{l}frac{alpha_i}{beta}x_iin A$ and $sum_{i={l+1}}^{k+1}frac{alpha_i}{gamma}x_iin A$. Call them $x$ and $y$ respectively.}



    Now, $sum_{i=1}^{k+1}alpha_ix_i = beta x+ gamma y$



    Since the result holds for $k=2$, $sum_{i=1}^{k+1}alpha_ix_iin A
    $





    My question is twofold. First, I never made use of the fact that $1+1neq0$ in F. What am I missing here? Second, how can I show that the condition in line 3 of my solution (when I introduce l and break the summation into two sums) can be assumed without loss of generality?



    Any input would be appreciated.










    share|cite|improve this question











    $endgroup$















      0












      0








      0





      $begingroup$


      The book I am using mentions this theorem:



      $$Asubseteq Vtext{ is a flat} iff Atext{ is closed under affine combinations, that is}\ x_1,ldots,x_kin A, alpha_1,ldots,alpha_k,in F, alpha_1+cdots+ alpha_k=1Rightarrow alpha_1x_1+cdots+alpha_kx_kin A$$



      The following question within the exercise then refers to this theorem (specifically the RHS of the above iff statement):




      Let $ 1+1neq0$ in $F$.



      Show that closure under affine combinations for $k=2$ implies closure under affine combinations for all $kgeq1$.






      My approach so far has been to use Strong Induction.



      Assuming the result holds for $kleq n$, we show it holds for $k=n+1$



      consider $sum_{i=1}^{k+1}alpha_ix_i $



      choose $lin{1,2,ldots, k+1}$ such that $sum_{i=1}^{k+1}alpha_ix_i=betasum_{i=1}^{l}frac{alpha_i}{beta}x_i+gammasum_{i={l+1}}^{k+1}frac{alpha_i}{gamma}x_i$



      (Note: $beta = sum_{i=1}^{l}alpha_i neq0$ and $gamma = sum_{i=l+1}^{k+1}alpha_i neq0$)



      Using induction hypothesis, $sum_{i=1}^{l}frac{alpha_i}{beta}x_iin A$ and $sum_{i={l+1}}^{k+1}frac{alpha_i}{gamma}x_iin A$. Call them $x$ and $y$ respectively.}



      Now, $sum_{i=1}^{k+1}alpha_ix_i = beta x+ gamma y$



      Since the result holds for $k=2$, $sum_{i=1}^{k+1}alpha_ix_iin A
      $





      My question is twofold. First, I never made use of the fact that $1+1neq0$ in F. What am I missing here? Second, how can I show that the condition in line 3 of my solution (when I introduce l and break the summation into two sums) can be assumed without loss of generality?



      Any input would be appreciated.










      share|cite|improve this question











      $endgroup$




      The book I am using mentions this theorem:



      $$Asubseteq Vtext{ is a flat} iff Atext{ is closed under affine combinations, that is}\ x_1,ldots,x_kin A, alpha_1,ldots,alpha_k,in F, alpha_1+cdots+ alpha_k=1Rightarrow alpha_1x_1+cdots+alpha_kx_kin A$$



      The following question within the exercise then refers to this theorem (specifically the RHS of the above iff statement):




      Let $ 1+1neq0$ in $F$.



      Show that closure under affine combinations for $k=2$ implies closure under affine combinations for all $kgeq1$.






      My approach so far has been to use Strong Induction.



      Assuming the result holds for $kleq n$, we show it holds for $k=n+1$



      consider $sum_{i=1}^{k+1}alpha_ix_i $



      choose $lin{1,2,ldots, k+1}$ such that $sum_{i=1}^{k+1}alpha_ix_i=betasum_{i=1}^{l}frac{alpha_i}{beta}x_i+gammasum_{i={l+1}}^{k+1}frac{alpha_i}{gamma}x_i$



      (Note: $beta = sum_{i=1}^{l}alpha_i neq0$ and $gamma = sum_{i=l+1}^{k+1}alpha_i neq0$)



      Using induction hypothesis, $sum_{i=1}^{l}frac{alpha_i}{beta}x_iin A$ and $sum_{i={l+1}}^{k+1}frac{alpha_i}{gamma}x_iin A$. Call them $x$ and $y$ respectively.}



      Now, $sum_{i=1}^{k+1}alpha_ix_i = beta x+ gamma y$



      Since the result holds for $k=2$, $sum_{i=1}^{k+1}alpha_ix_iin A
      $





      My question is twofold. First, I never made use of the fact that $1+1neq0$ in F. What am I missing here? Second, how can I show that the condition in line 3 of my solution (when I introduce l and break the summation into two sums) can be assumed without loss of generality?



      Any input would be appreciated.







      linear-algebra






      share|cite|improve this question















      share|cite|improve this question













      share|cite|improve this question




      share|cite|improve this question








      edited Dec 7 '18 at 20:02









      Henning Makholm

      239k17304541




      239k17304541










      asked Dec 7 '18 at 19:07









      s0ulr3aper07s0ulr3aper07

      1499




      1499






















          1 Answer
          1






          active

          oldest

          votes


















          2












          $begingroup$

          The assumption about the characteristic of $F$ comes into play when we justify the claim you ask about, so the questions are one in the same. Here's a start...



          Firstly note that you can assume all the $alpha_i$ are nonzero.

          Then start by dividing the sum into $beta' = alpha_n$ and $gamma' = sum_{i<n} alpha_i$. We split the problem into cases.



          Case (1) $gamma' not = 0$ and there is nothing to prove.



          Case (2) $gamma' = 0$ and $alpha_j not= -alpha_n$ for some $j <n$. Then we modify the division as $beta = alpha_n + a_j$ and $gamma = sum_{i not= j,n}alpha_i$ and are done.



          Case (3) $gamma' = 0$ and $alpha_j = -alpha_n$ for all $j <n$. By assumption, $2alpha_j not= 0$. Hence, if we set $$gamma = gamma' + 2alpha_n = sum_{j< n-2} alpha_j$$ $$beta = beta' - 2alpha_n = sum_{igeq n-2} alpha_i = -alpha_n$$ then $beta, gamma not= 0$






          share|cite|improve this answer











          $endgroup$













          • $begingroup$
            If $gamma = 0$, then $beta = alpha_n = 1$. Going to case (3) that you've mentioned, it would imply that $gamma = 0=-alpha_n-alpha_n-cdots -alpha_n=-1-1-cdots - 1$. Now how would I proceed from this point and use $1+1neq 0$?
            $endgroup$
            – s0ulr3aper07
            Dec 7 '18 at 22:18






          • 1




            $begingroup$
            i edited a better explanation for case (3), hope that helps
            $endgroup$
            – Badam Baplan
            Dec 7 '18 at 23:04











          Your Answer





          StackExchange.ifUsing("editor", function () {
          return StackExchange.using("mathjaxEditing", function () {
          StackExchange.MarkdownEditor.creationCallbacks.add(function (editor, postfix) {
          StackExchange.mathjaxEditing.prepareWmdForMathJax(editor, postfix, [["$", "$"], ["\\(","\\)"]]);
          });
          });
          }, "mathjax-editing");

          StackExchange.ready(function() {
          var channelOptions = {
          tags: "".split(" "),
          id: "69"
          };
          initTagRenderer("".split(" "), "".split(" "), channelOptions);

          StackExchange.using("externalEditor", function() {
          // Have to fire editor after snippets, if snippets enabled
          if (StackExchange.settings.snippets.snippetsEnabled) {
          StackExchange.using("snippets", function() {
          createEditor();
          });
          }
          else {
          createEditor();
          }
          });

          function createEditor() {
          StackExchange.prepareEditor({
          heartbeatType: 'answer',
          autoActivateHeartbeat: false,
          convertImagesToLinks: true,
          noModals: true,
          showLowRepImageUploadWarning: true,
          reputationToPostImages: 10,
          bindNavPrevention: true,
          postfix: "",
          imageUploader: {
          brandingHtml: "Powered by u003ca class="icon-imgur-white" href="https://imgur.com/"u003eu003c/au003e",
          contentPolicyHtml: "User contributions licensed under u003ca href="https://creativecommons.org/licenses/by-sa/3.0/"u003ecc by-sa 3.0 with attribution requiredu003c/au003e u003ca href="https://stackoverflow.com/legal/content-policy"u003e(content policy)u003c/au003e",
          allowUrls: true
          },
          noCode: true, onDemand: true,
          discardSelector: ".discard-answer"
          ,immediatelyShowMarkdownHelp:true
          });


          }
          });














          draft saved

          draft discarded


















          StackExchange.ready(
          function () {
          StackExchange.openid.initPostLogin('.new-post-login', 'https%3a%2f%2fmath.stackexchange.com%2fquestions%2f3030260%2fquestion-on-linear-algebra-related-to-affine-combination%23new-answer', 'question_page');
          }
          );

          Post as a guest















          Required, but never shown

























          1 Answer
          1






          active

          oldest

          votes








          1 Answer
          1






          active

          oldest

          votes









          active

          oldest

          votes






          active

          oldest

          votes









          2












          $begingroup$

          The assumption about the characteristic of $F$ comes into play when we justify the claim you ask about, so the questions are one in the same. Here's a start...



          Firstly note that you can assume all the $alpha_i$ are nonzero.

          Then start by dividing the sum into $beta' = alpha_n$ and $gamma' = sum_{i<n} alpha_i$. We split the problem into cases.



          Case (1) $gamma' not = 0$ and there is nothing to prove.



          Case (2) $gamma' = 0$ and $alpha_j not= -alpha_n$ for some $j <n$. Then we modify the division as $beta = alpha_n + a_j$ and $gamma = sum_{i not= j,n}alpha_i$ and are done.



          Case (3) $gamma' = 0$ and $alpha_j = -alpha_n$ for all $j <n$. By assumption, $2alpha_j not= 0$. Hence, if we set $$gamma = gamma' + 2alpha_n = sum_{j< n-2} alpha_j$$ $$beta = beta' - 2alpha_n = sum_{igeq n-2} alpha_i = -alpha_n$$ then $beta, gamma not= 0$






          share|cite|improve this answer











          $endgroup$













          • $begingroup$
            If $gamma = 0$, then $beta = alpha_n = 1$. Going to case (3) that you've mentioned, it would imply that $gamma = 0=-alpha_n-alpha_n-cdots -alpha_n=-1-1-cdots - 1$. Now how would I proceed from this point and use $1+1neq 0$?
            $endgroup$
            – s0ulr3aper07
            Dec 7 '18 at 22:18






          • 1




            $begingroup$
            i edited a better explanation for case (3), hope that helps
            $endgroup$
            – Badam Baplan
            Dec 7 '18 at 23:04
















          2












          $begingroup$

          The assumption about the characteristic of $F$ comes into play when we justify the claim you ask about, so the questions are one in the same. Here's a start...



          Firstly note that you can assume all the $alpha_i$ are nonzero.

          Then start by dividing the sum into $beta' = alpha_n$ and $gamma' = sum_{i<n} alpha_i$. We split the problem into cases.



          Case (1) $gamma' not = 0$ and there is nothing to prove.



          Case (2) $gamma' = 0$ and $alpha_j not= -alpha_n$ for some $j <n$. Then we modify the division as $beta = alpha_n + a_j$ and $gamma = sum_{i not= j,n}alpha_i$ and are done.



          Case (3) $gamma' = 0$ and $alpha_j = -alpha_n$ for all $j <n$. By assumption, $2alpha_j not= 0$. Hence, if we set $$gamma = gamma' + 2alpha_n = sum_{j< n-2} alpha_j$$ $$beta = beta' - 2alpha_n = sum_{igeq n-2} alpha_i = -alpha_n$$ then $beta, gamma not= 0$






          share|cite|improve this answer











          $endgroup$













          • $begingroup$
            If $gamma = 0$, then $beta = alpha_n = 1$. Going to case (3) that you've mentioned, it would imply that $gamma = 0=-alpha_n-alpha_n-cdots -alpha_n=-1-1-cdots - 1$. Now how would I proceed from this point and use $1+1neq 0$?
            $endgroup$
            – s0ulr3aper07
            Dec 7 '18 at 22:18






          • 1




            $begingroup$
            i edited a better explanation for case (3), hope that helps
            $endgroup$
            – Badam Baplan
            Dec 7 '18 at 23:04














          2












          2








          2





          $begingroup$

          The assumption about the characteristic of $F$ comes into play when we justify the claim you ask about, so the questions are one in the same. Here's a start...



          Firstly note that you can assume all the $alpha_i$ are nonzero.

          Then start by dividing the sum into $beta' = alpha_n$ and $gamma' = sum_{i<n} alpha_i$. We split the problem into cases.



          Case (1) $gamma' not = 0$ and there is nothing to prove.



          Case (2) $gamma' = 0$ and $alpha_j not= -alpha_n$ for some $j <n$. Then we modify the division as $beta = alpha_n + a_j$ and $gamma = sum_{i not= j,n}alpha_i$ and are done.



          Case (3) $gamma' = 0$ and $alpha_j = -alpha_n$ for all $j <n$. By assumption, $2alpha_j not= 0$. Hence, if we set $$gamma = gamma' + 2alpha_n = sum_{j< n-2} alpha_j$$ $$beta = beta' - 2alpha_n = sum_{igeq n-2} alpha_i = -alpha_n$$ then $beta, gamma not= 0$






          share|cite|improve this answer











          $endgroup$



          The assumption about the characteristic of $F$ comes into play when we justify the claim you ask about, so the questions are one in the same. Here's a start...



          Firstly note that you can assume all the $alpha_i$ are nonzero.

          Then start by dividing the sum into $beta' = alpha_n$ and $gamma' = sum_{i<n} alpha_i$. We split the problem into cases.



          Case (1) $gamma' not = 0$ and there is nothing to prove.



          Case (2) $gamma' = 0$ and $alpha_j not= -alpha_n$ for some $j <n$. Then we modify the division as $beta = alpha_n + a_j$ and $gamma = sum_{i not= j,n}alpha_i$ and are done.



          Case (3) $gamma' = 0$ and $alpha_j = -alpha_n$ for all $j <n$. By assumption, $2alpha_j not= 0$. Hence, if we set $$gamma = gamma' + 2alpha_n = sum_{j< n-2} alpha_j$$ $$beta = beta' - 2alpha_n = sum_{igeq n-2} alpha_i = -alpha_n$$ then $beta, gamma not= 0$







          share|cite|improve this answer














          share|cite|improve this answer



          share|cite|improve this answer








          edited Dec 7 '18 at 23:02

























          answered Dec 7 '18 at 20:01









          Badam BaplanBadam Baplan

          4,501722




          4,501722












          • $begingroup$
            If $gamma = 0$, then $beta = alpha_n = 1$. Going to case (3) that you've mentioned, it would imply that $gamma = 0=-alpha_n-alpha_n-cdots -alpha_n=-1-1-cdots - 1$. Now how would I proceed from this point and use $1+1neq 0$?
            $endgroup$
            – s0ulr3aper07
            Dec 7 '18 at 22:18






          • 1




            $begingroup$
            i edited a better explanation for case (3), hope that helps
            $endgroup$
            – Badam Baplan
            Dec 7 '18 at 23:04


















          • $begingroup$
            If $gamma = 0$, then $beta = alpha_n = 1$. Going to case (3) that you've mentioned, it would imply that $gamma = 0=-alpha_n-alpha_n-cdots -alpha_n=-1-1-cdots - 1$. Now how would I proceed from this point and use $1+1neq 0$?
            $endgroup$
            – s0ulr3aper07
            Dec 7 '18 at 22:18






          • 1




            $begingroup$
            i edited a better explanation for case (3), hope that helps
            $endgroup$
            – Badam Baplan
            Dec 7 '18 at 23:04
















          $begingroup$
          If $gamma = 0$, then $beta = alpha_n = 1$. Going to case (3) that you've mentioned, it would imply that $gamma = 0=-alpha_n-alpha_n-cdots -alpha_n=-1-1-cdots - 1$. Now how would I proceed from this point and use $1+1neq 0$?
          $endgroup$
          – s0ulr3aper07
          Dec 7 '18 at 22:18




          $begingroup$
          If $gamma = 0$, then $beta = alpha_n = 1$. Going to case (3) that you've mentioned, it would imply that $gamma = 0=-alpha_n-alpha_n-cdots -alpha_n=-1-1-cdots - 1$. Now how would I proceed from this point and use $1+1neq 0$?
          $endgroup$
          – s0ulr3aper07
          Dec 7 '18 at 22:18




          1




          1




          $begingroup$
          i edited a better explanation for case (3), hope that helps
          $endgroup$
          – Badam Baplan
          Dec 7 '18 at 23:04




          $begingroup$
          i edited a better explanation for case (3), hope that helps
          $endgroup$
          – Badam Baplan
          Dec 7 '18 at 23:04


















          draft saved

          draft discarded




















































          Thanks for contributing an answer to Mathematics Stack Exchange!


          • Please be sure to answer the question. Provide details and share your research!

          But avoid



          • Asking for help, clarification, or responding to other answers.

          • Making statements based on opinion; back them up with references or personal experience.


          Use MathJax to format equations. MathJax reference.


          To learn more, see our tips on writing great answers.




          draft saved


          draft discarded














          StackExchange.ready(
          function () {
          StackExchange.openid.initPostLogin('.new-post-login', 'https%3a%2f%2fmath.stackexchange.com%2fquestions%2f3030260%2fquestion-on-linear-algebra-related-to-affine-combination%23new-answer', 'question_page');
          }
          );

          Post as a guest















          Required, but never shown





















































          Required, but never shown














          Required, but never shown












          Required, but never shown







          Required, but never shown

































          Required, but never shown














          Required, but never shown












          Required, but never shown







          Required, but never shown







          Popular posts from this blog

          Quarter-circle Tiles

          build a pushdown automaton that recognizes the reverse language of a given pushdown automaton?

          Mont Emei